4. Area and Average Value

Exercises

    In each of the following problems, find the area under the graph of \(f\), above the \(x\)-axis, between \(x=a\) and \(x=b\).

  1. \(f(x)=-x^2+4x-2\),   \(a=1\) and \(b=3\).

    x_below_x^2+4x-2_rect_anim

    Remember the area is the area under the curve: \[ A=\int_a^b f(x)\,dx \]

    \(A=\dfrac{10}{3}\)

    The area is \[\begin{aligned} A&=\int_1^3 (-x^2+4x-2)\,dx \\ &=\left[-\dfrac{x^3}{3}+2x^2-2x\right]_1^3 \\ &=(-9+18-6)-\left(-\dfrac{1}{3}+2-2\right) =\dfrac{10}{3} \end{aligned}\]

    jm,db 

  2. \(f(x)=x^3-2x\),   \(a=0\) and \(b=4\).

    \(\displaystyle A=48\)

    The area is \[\begin{aligned} A&=\int_0^4 x^3-2x\,dx =\left[\dfrac{x^4}{4}-x^2\right]_0^4 \\ &=\left[\dfrac{4^4}{4}-4^2\right]-0 =64-16=48 \end{aligned}\]

  3. \(f(x)=e^x+1\),   \(a=-1\) and \(b=2\).

    \(A=e^2-\dfrac{1}{e}+3\)

    The area is \[\begin{aligned} A&=\int_{-1}^{2} e^x + 1\,dx =\left[\dfrac{}{}e^x + x\right]_{-1}^{2} \\ &=\left[(e^2 + 2)-\left(\dfrac{1}{e}-1\right)\right] \\ &=e^2 -\dfrac{1}{e} + 3 \end{aligned}\]

    x_below_e^x_rect_anim

    db 

  4. \(f(x)=x(x^2+1)^{5}\),   \(a=0\) and \(b=1\).

    You'll need to use a substitution to do the integral.

    \(\displaystyle A=\dfrac{21}{4}\)

    The area is the integral: \[ A=\int_0^1 x(x^2+1)^{5}\,dx \] To do the integral, we make the substitution \(u=x^2+1\). Then \(du=2x\,dx\) and \(x\,dx=\dfrac{1}{2}\,du\). So the area is: \[ A=\dfrac{1}{2}\int_1^2 u^{5}du =\dfrac{1}{2}\left[\dfrac{u^{6}}{6}\right]_1^2 =\dfrac{2^6-1}{12} =\dfrac{21}{4} \]

    x_below_x(x^2+1)_rect_anim

    jm,db 


  5. Challenge: The total sales of widgets sold since they went on the market is given by the function \(G(t)\) where \(t\) is time in days. So the rate they are being sold is \(G'(t)\). Explain in practical terms what the area of the region \(R\) represents where \(R\) is the region below the graph of \(G'(t)\), above the horizontal axis and between times \(t_1\)and \(t_2\).

    Recall the Fundamental Theorem of the Integral Calculus.

    By the Fundamental Theorem: \[ \int_{t_1}^{t_2} G'(t)dt=G(t_2)-G(t_1) \] The left side of this equation is the area of \(R\). The right side is the difference between total sales by day \(t_2\) and by day \(t_1\). So the area represents the sales between day \(t_1\) and day \(t_2\).

    jm 

  6. In each of the following problems, sketch the region between the graphs of \(f\) and \(g\) and find its area.

  7. \(f(x)=x^3\) and \(g(x)=x^2\)

    To find where they intersect, we equate them.
    To see which is on top, evaluate each function at \(x=\dfrac{1}{2}\).

    \(A=\dfrac{1}{12}\)

    x_betw_x^3_x^2

    To find where they intersect, we equate them: \[ x^3=x^2 \quad \Rightarrow \quad x^3-x^2=0 \quad \Rightarrow \quad x^2(x-1)=0 \] They intersect at \(x=0\) and \(x=1\). To see which is on top, we evaluate at \(x=\dfrac{1}{2}\): \[ f\left(\dfrac{1}{2}\right)=\left(\dfrac{1}{2}\right)^3=\dfrac{1}{8} \qquad g\left(\dfrac{1}{2}\right)=\left(\dfrac{1}{2}\right)^2=\dfrac{1}{4} \] So \(g(x)=x^2\) is bigger and on top. So the area is: \[ A=\int_0^1 x^2-x^3\,dx =\left[\dfrac{x^3}{3}-\dfrac{x^4}{4}\right]_0^1 =\dfrac{1}{3}-\dfrac{1}{4} =\dfrac{1}{12} \]

    betwexp

    jm,db 

  8. \(f(x)=x-1\) and \(g(x)=x^2-4x+3\)

    \(A=\dfrac{9}{2}\)

    x_betw_x-1_x^2-4x+3

    We first find the points of intersection: \[\begin{aligned} x-1&=x^2-4x+3 \\ 0&=x^2-5x+4=(x-4)(x-1) \\ &\Rightarrow x=1,4 \end{aligned}\] Since \(f\) is the upper function, we evaluate: \[\begin{aligned} \int_1^4 &[(x-1)-(x^2-4x+3)]\,dx =\int_1^4 (-x^2+5x-4)\,dx \\ &=\left[-\dfrac{x^3}{3}+\dfrac{5x^2}{2}-4x\right]_1^4 \\ &=-\dfrac{64}{3}+40-16+\dfrac{1}{3}-\dfrac{5}{2}+4 =\dfrac{9}{2} \end{aligned}\]

    x_betw_x-1_x^2-4x+3

    jm,db 

  9. \(f(x)=x^3-4x\) and \(g(x)=5x\)

    \(A=\dfrac{81}{2}\)

    x_betw_x^3-4x_5x

    We first solve for the intersection points: \[\begin{aligned} &x^3-4x=5x \quad \Rightarrow \quad x^3-9x=0 \\ &\Rightarrow \quad x(x-3)(x+3)=0 \quad \Rightarrow \quad x=-3,0,3 \end{aligned}\] So we need to break the integral into two pieces. From the graph we see that the cubic is on top for the first integral while the line is on top for the second integral. So the area is:

    x_betw_x^3-4x_5x

    \[\begin{aligned} A&=\int_{-3}^0 \text{upper}-\text{lower}\,dx +\int_0^3 \text{upper}-\text{lower}\,dx \\ &=\int_{-3}^0 [(x^3-4x)-5x]\,dx+\int_0^3 [5x-(x^3-4x)]\,dx \\ &=\int_{-3}^0 [x^3-9x]\,dx+\int_0^3 [9x-x^3]\,dx \\ &=\left[\dfrac{x^4}{4}-9\dfrac{x^2}{2}\right]_{-3}^0 +\left[ 9\dfrac{x^2}{2}-\dfrac{x^4}{4}\right]_0^3 \\ &=[ 0]-\left[\dfrac{81}{4}-9\dfrac{9}{2}\right] +\left[ 9\dfrac{9}{2}-\dfrac{81}{4}\right]-[ 0] =\dfrac{81}{2} \end{aligned}\]

    jm 

    Since the region is symmetric, we could have doubled the area of one piece: \[ A=2\int_0^3 [ 5x-(x^3-4x)]\,dx \]


  10. In each of the following problems, sketch the region bounded by the given curves and find its area.

  11. \(y=e^x\), \(y=e^{-x}\) and \(x=2\)

    Notice this region is to the right of both exponentials.

    \(A=e^2+e^{-2}-2\)

    x_in_e^x_e^-x_x=2

    The exponentials intersect when \(e^x=e^{-x}\) or \(e^{2x}=1\) or \(x=0\). So the area is: \[\begin{aligned} A&=\int_0^2 e^x-e^{-x}\,dx =[e^x+e^{-x}]_0^2 \\ &=[e^2+e^{-2}]-[e^0+e^0] =e^2+e^{-2}-2 \end{aligned}\]

    x_in_e^x_e^-x_x=2

    jm,db 

  12. \(y=e^x\), \(y=e^{-x}\) and \(y=2\)

    Notice this region is above both exponentials and is symmetric about the \(y\)-axis.

    \(A=4\ln 2-2\)

    x_in_e^x_e^-x_y=2

    By symmetry, we will double the area to the right of \(x=0\). The line \(y=2\) intersects the exponential \(y=e^x\) at \(x=\ln2\). So: \[\begin{aligned} A&=2\int_0^{\ln 2} (2-e^x)dx =2\left[\dfrac{}{}2x-e^x\right]_0^{\ln 2} \\ &=2[ 2\ln 2-e^{\ln 2}-(0-1)] \\ &=2[ 2\ln 2-2+1] =4\ln 2-2 \end{aligned}\]

    x_in_e^x_e^-x_y=2

    jm,db 


  13. In each problem, sketch the region(s) bounded by the graph of \(f\) and the \(x\)-axis. Then compute its area.

  14. \(f(x)=x^3+x^2-6x\)

    Consider this the area between 2 curves, one the graph of \(f\) and the other the \(x\)-axis.

    \(A=\dfrac{253}{12}\)

    x_betw_x^3+x^2-6x_x-axis

    We first find x-intercepts: \[ x^3+x^2-6x=x(x-2)(x+3)=0 \] \[ x=-3,0,2 \] From the graph we see that the cubic is positive for the first integral and negative for the second one:

    x_betw_x^3+x^2-6x_x-axis

    \[\begin{aligned} A&=\int_{-3}^0 \text{upper}-\text{lower}\,dx +\int_0^2 \text{upper}-\text{lower}\,dx \\ &=\int_{-3}^0 [(x^3+x^2-6x)-0]\,dx+\int_0^2 [0-(x^3+x^2-6x)]\,dx \\ &=\dfrac{63}{4}+\dfrac{16}{3}=\dfrac{253}{12} \end{aligned}\]

    jm,db 

  15. In each problem, sketch the region(s) bounded by the graph of \(f\) and the \(y\)-axis. Then compute its area.

  16. \(x=\sin y\)   for \(0 \le y \le \pi\)

    \(A=2\)

    The area is: \[\begin{aligned} A&=\int_0^\pi\sin y\,dy =\left[-\cos y\rule{0pt}{12pt}\right]_0^\pi \\ &=-(-1)-(-1)=2 \end{aligned}\]

    x_siny_0_to_Pi

    jm,db 

  17. \(x=16-y^4\)

    \(A=\dfrac{256}{5}\)

    We find the endpoints: \[ 16-y^4=0 \quad \Rightarrow \quad y=\pm2 \] \[\begin{aligned} A&=\int_{-2}^2 (16-y^4)\,dy =\left[16y-\dfrac{y^5}{5}\right]_{-2}^2 \\ &=2\left(32-\dfrac{32}{5}\right) =\dfrac{256}{5} \end{aligned}\]

    x_16-y^4_-2_to_2

    jm,db 


  18. In each of the following problems, sketch the region bounded by the graphs of the given functions and find its area.

  19. \(y^2=1+x\) and the line joining \((-1,0)\) and \((0,1)\)

    Write the edges with \(x\) as a function of \(y\). What are the endpoints? Which curve is on the right (larger \(x\))?

    \(A=\dfrac{1}{6}\)

    As functions of \(y\) the edges are the parabola \(x=y^2-1\) and the line \(x=y-1\). From the plot we see they intersect at \(y=0\) and \(y=1\). However, if we do not have a plot, we can find where they intersect by equating them: \[ y^2-1=y-1 \quad \text{or} \quad y=0,1 \] From the plot we see that the line is on the right and the parabola is on the left. However, if we do not have a plot, we can plug in \(y=\dfrac{1}{2}\) into the parabola to get \(x=-\dfrac{3}{4}\) and into the line to get \(x=-\dfrac{1}{2}\). So the line is on the right.

    x_y^2-1_0_to_1

    Then the area is: \[\begin{aligned} A&=\int_0^1 (y-1)-(y^2-1)\,dy =\int_0^1 (y-y^2)\,dy \\ &=\left[\dfrac{y^2}{2}-\dfrac{y^3}{3}\right]_0^1 =\dfrac{1}{2}-\dfrac{1}{3}=\dfrac{1}{6} \end{aligned}\]

    jm,db 

  20. \(x=y^3-9y\) and the \(y\)-axis.

    What are the intersection points? When is \(x \gt 0\)? When \(x \lt 0\)?

    \(A=\dfrac{81}{2}\).

    The functions intersect when \(0=y^3-9y=y(y^2-9)\) or \(y=-3,0,3\). When we plug in \(y=1\), we get \(x=-8\). When we plug in \(y=-1\), we get \(x=8\). So the curve is to the left of the \(y\)-axis for \(y \ge 0\) and to the right of the \(y\)-axis for \(y \le 0\). So the area is: \[\begin{aligned} A&=\int_{-3}^0 (y^3-9y)-(0)\,dy+\int_0^3 (0)-(y^3-9y)\,dy \\ &=\left[\dfrac{y^4}{4}-\dfrac{9y^2}{2}\right]_{-3}^0 +\left[-\dfrac{y^4}{4}+\dfrac{9y^2}{2}\right]_0^3 \\ &=2\left(-\dfrac{81}{4}+\dfrac{81}{2}\right) =\dfrac{81}{2} \end{aligned}\]

    jm 

  21. In each of the following problems, find the average value of the given function on the given interval. Plot the function and its average.

  22. \(f(x)=x^3\) on \([-1,2]\)

    x_ave_cubic_prob

    \(f_\text{ave}=\dfrac{5}{4}\)

    x_ave_cubic_sol

    \[\begin{aligned} f_\text{ave} &=\dfrac{1}{2-(-1)}\int_{-1}^2 x^3\,dx =\dfrac{1}{3}\left[ \dfrac{x^4}{4}\right]_{-1}^2 \\ &=\dfrac{1}{12}(16-1) =\dfrac{5}{4} \end{aligned}\]

    x_ave_cubic_sol

    jm,py 

  23. \(f(x)=x^4-x^2\) on \([ 0,2]\)

    x_ave_quartic_prob

    \(f_\text{ave}=\dfrac{28}{15}\)

    x_ave_quartic_sol

    \[\begin{aligned} f_\text{ave} &=\dfrac{1}{2}\int_0^2 x^4-x^2\,dx =\dfrac{1}{2}\left[\dfrac{x^5}{5}-\dfrac{x^3}{3}\right]_0^2 \\ &=\dfrac{1}{2}\left(\dfrac{32}{5}-\dfrac{8}{3}\right)=\dfrac{28}{15} \end{aligned}\]

    x_ave_quartic_sol

    jm,py 


  24. The identity \(\sin^2 x=\dfrac{1-\cos(2x)}{2}\) allows us to do the following integrals. See the chapter on Trig Integrals.
    1. Compute \(\displaystyle \int \sin^2 x\,dx\).

      \(\displaystyle \int \sin^2 x\,dx =\dfrac{1}{2}\left(x-\dfrac{\sin(2x)}{2}\right)+C \)

      \[\begin{aligned} \int \sin^2 x\,dx &=\dfrac{1}{2}\int [1-\cos(2x)]\,dx \\ &=\dfrac{1}{2}\left(x-\dfrac{\sin(2x)}{2}\right)+C \end{aligned}\]

      jm 

      If   \(f(x)=\dfrac{1}{2}\left(x-\dfrac{\sin(2x)}{2}\right)\) then: \[ \dfrac{df}{dx} =\dfrac{1}{2}\left(1-\cos(2x)\right) =\sin^2 x \]

    2. Find the average value of \(\sin^2 x\) on \(\left[0,\dfrac{\pi}{4}\right]\).

      The average value of \(f\) on \(\left[0,\dfrac{\pi}{4}\right]\) is \(\displaystyle f_\text{ave}=\dfrac{4}{\pi}\int_0^{\pi/4} f(x)\,dx\).

      The average value of \(\sin^2 x\) on \(\left[0,\dfrac{\pi}{4}\right]\) is \(\dfrac{1}{2}-\dfrac{1}{\pi}\).

      If \(f=\sin^2 x\), then \[\begin{aligned} f_\text{ave} &=\dfrac{4}{\pi}\int_0^{\pi/4} \sin^2 x\,dx \\ &=\dfrac{4}{\pi}\dfrac{1}{2}\left[x-\dfrac{\sin(2x)}{2}\right]_0^{\pi/4} \\ &=\dfrac{2}{\pi}\left(\dfrac{\pi}{4}-\dfrac{\sin\dfrac{\pi}{2}}{2}\right) =\dfrac{1}{2}-\dfrac{1}{\pi} \end{aligned}\]

      jm 

    3. Find the average value of \(\sin^2 x\) on \([0,2\pi]\).

      The average value of \(f\) on \([0,2\pi]\) is \(\displaystyle f_\text{ave}=\dfrac{1}{2\pi}\int_0^{2\pi} f(x)\,dx\).

      The average value of \(\sin^2 x\) on \([0,2\pi]\) is \(\dfrac{1}{2}\).

      If \(f=\sin^2 x\), then \[\begin{aligned} f_\text{ave} &=\dfrac{1}{2\pi}\int_0^{2\pi} \sin^2 x\,dx \\ &=\dfrac{1}{2\pi}\dfrac{1}{2}\left[x-\dfrac{\sin(2x)}{2}\right]_0^{2\pi} \\ &=\dfrac{1}{4\pi}\left(2\pi-\dfrac{\sin4\pi}{2}\right) =\dfrac{1}{2} \end{aligned}\]

      jm 

      This should not be surprising. If we look at the graph of \(\sin^2 x\), the center line is \(y=\dfrac{1}{2}\).

      x_ave_sin^2_sol

      py 


  25. In each of the following problems, find the height of a rectangle whose base is the given interval on the \(x\)-axis and whose area matches that of the region below the given function on the given interval. Plot the function and the rectangle.

  26. \(y=e^x\) on \([0,2]\)

    The height is the average value of the function.

    \(y_\text{ave}=\dfrac{e^2-1}{2}\)

    x_ave_e^x

    \[\begin{aligned} y_\text{ave} &=\dfrac{1}{2}\int_0^2 e^x\,dx =\dfrac{1}{2}\left[e^x\rule{0pt}{12pt}\right]_0^2 \\ &=\dfrac{e^2-1}{2} \approx3.2 \end{aligned}\] Notice the area of the rectangle is the same as the area below the curve.

    x_ave_e^x

    jm,py 

  27. \(f(x)=x^4-4x^2+4\) on \([-2,2]\)

    The height is the average value of the function.

    \(f_\text{ave}=\dfrac{28}{15}\)

    x_ave_x^4-4x^2+4

    \[\begin{aligned} f_\text{ave} &=\dfrac{1}{2-(-2)}\int_{-2}^2 x^4-4x^2+4\,dx \\ &=\dfrac{1}{4}\left[\dfrac{x^5}{5}-\dfrac{4x^3}{3}+4x\right]_{-2}^2 \\ &=2\cdot\dfrac{1}{4}\left(\dfrac{2^5}{5}-\dfrac{4\cdot2^3}{3}+4\cdot2\right) \\ &=\dfrac{2^4}{5}-\dfrac{4\cdot2^2}{3}+4 \\ &=\dfrac{48-80+60}{15}=\dfrac{28}{15} \end{aligned}\]

    x_ave_x^4-4x^2+4

    jm,py 


  28. In each of the following problems, find the value(s) of \(c\) guaranteed by the Mean Value Theorem for Integrals for the given function on the given interval.

  29. \(f(x)=e^x\) on \([ 0,2]\)

    \(f(c)=f_\text{ave}\)

    \(c=\ln\dfrac{e^2-1}{2}\approx1.16\)

    x_mvt_e^x

    Since \(f_\text{ave}=\dfrac{e^2-1}{2}\), (See a previous exercise.) we solve \[ f(c)=e^c=f_\text{ave}=\dfrac{e^2-1}{2} \] to get: \[ c=\ln\dfrac{e^2-1}{2}\approx1.16 \]

    x_mvt_e^x

    jm,py 

  30. \(f(x)=x^4\) on \([ 0,5]\)

    \(f(c)=f_\text{ave}\)

    \(c=5^{3/4}\approx3.34\)

    We first find the average: \[ f_\text{ave} =\dfrac{1}{5}\int_0^5 x^4\,dx =\left[\dfrac{1}{25}x^5\right]_0^5 =125 \] Then we solve \(f(c)=f_\text{ave}\): \[ c^4=125 \quad \Rightarrow \quad c=5^{3/4}\approx3.34 \]

    jm 

  31. Review Exercises

  32. Find the area of the region above \(y=\cos x\), below \(y=1\), for \(0 \le x \le \pi\).

    \(A=\pi\)

    \[ A=\int_0^\pi 1-\cos x\,dx =\left[x-\sin x\right]_0^\pi =\pi-\sin\pi =\pi \]

    jm 

    This should not be surprising. If we look at the graph of \(\cos x\), we see the required area is half of the area of the rectangle, \(0 \le x \le \pi\) and \(-1 \le y \le 1\), whose area is \(2\pi\).

    x_area_cos_to_1

    py 


  33. Find the area of the region bounded by the given curves and lines.

  34. \(x=1-y^{2}\), \(y\)-axis

    Is this an \(x\)-integral or a \(y\)-integral?

    \(A = \dfrac{4}{3}\)

    The curve \(x=1-y^{2}\) intersects the \(y\)-axis at \(y=\pm1\). So the area is: \[\begin{aligned} A&=\int_{-1}^1 (1-y^2) \,dy =\left[y-\dfrac{y^{3}}{3}\right]_{-1}^1 \\ &=1-\dfrac{1^3}{3}-\left(-1-\dfrac{(-1)^3}{3}\right) =2(1-\dfrac{1}{3}) =\dfrac{4}{3} \end{aligned}\]

    jr 

  35. \(y=x^{3}-3x^{2}\), \(x\)-axis

    Where do the function intersect? Which function is bigger in each piece?

    \(A = \dfrac{27}{4}\)

    Let \(f(x)=x^{3}-3x^{2}\) and \(g(x)=0\). We first find where the curves intersect: \[ x^{3}-3x^{2}=0 \implies x=0,3 \] To see which is bigger, we try a point in between, \(x=1\): \[ f(1)=1^{3}-3\cdot1^{2}=-2 \quad\text{and}\quad g(1)=0 \] So \(g(x)=0\) is on top. This is confirmed in a plot.

    x_x^3-3x_x^2

    So the area is: \[\begin{aligned} A&=\int_0^3 g(x)-f(x)\,dx =\int_0^3 -x^3+3x^2 \,dx \\ &=\left[-\,\dfrac{x^4}{4}+x^3\right]_0^3 =-\,\dfrac{81}{4}+27 =-\,\dfrac{81}{4}+\dfrac{108}{4} =\dfrac{27}{4} \end{aligned}\]

    jr 

  36. \(y=x^{3}-3x\), \(x\)-axis

    Where do the function intersect? Which function is bigger in each piece?

    \(A = \dfrac{9}{2}\)

    Let \(f(x)=x^{3}-3x\) and \(g(x)=0\). We first find where the curves intersect: \[ x^{3}-3x=0 \implies x=0,\pm\sqrt 3 \] We find which is on top or bottom by evaluating \(f\) at a number in each interval: \[\begin{aligned} f(-1)&=-1+3=2 \gt 0 \\ f(1)&=1-3=-2 \lt 0 \end{aligned}\] So the \(f\) is on the top from \(-\sqrt3\) to \(0\) and on the bottom from \(0\) to \(\sqrt3\). This is confirmed in a plot.

    x_x^3-3x_x

    So the area is: \[\begin{aligned} A&=\int_{-\sqrt 3}^0 (f(x)-g(x))\,dx+\int_0^{\sqrt 3} (g(x)-f(x))\,dx \\ &=\int_{-\sqrt 3}^0 (x^3-3x)\,dx+\int_0^{\sqrt 3} (-x^3+3x)\,dx \\ &=\left[\dfrac{x^4}{4}-\dfrac{3x^2}{2}\right]_{-\sqrt 3}^0 +\left[-\,\dfrac{x^4}{4}+\dfrac{3x^2}{2}\right]_0^{\sqrt 3} \\ &=-\left(\dfrac{9}{4}-\dfrac{9}{2}\right)+\left(-\dfrac{9}{4}+\dfrac{9}{2}\right) =2\left(\dfrac{9}{4}\right) = \dfrac{9}{2} \end{aligned}\] Note: We could have doubled either one of these integrals since the region is symmetric.

    jr 

© MYMathApps

Supported in part by NSF Grant #1123255